Showing a sum is positiveFinding Binomial expansion of a radicalSimplify the Expression $sum _ k=0 ^ n binomnki^k3^k-n $Alternative combinatorial proof for $sumlimits_r=0^nbinomnrbinomm+rn=sumlimits_r=0^nbinomnrbinommr2^r$Proof by induction, binomial coefficientApproximating a binomial sum over a simplexHow to expand $sqrtx^6+1$ using Maclaurin's seriesSum of $m choose j$ multiplied by $2^2^j$How to show that $sumlimits_k=0^n (-1)^ktfrac nchoosek x+kchoosek = fracxx+n$Finite sum with inverse binomialShowing an alternating sum is positive

Why should universal income be universal?

Does the reader need to like the PoV character?

What are some good ways to treat frozen vegetables such that they behave like fresh vegetables when stir frying them?

Creating two special characters

What is going on with gets(stdin) on the site coderbyte?

I found an audio circuit and I built it just fine, but I find it a bit too quiet. How do I amplify the output so that it is a bit louder?

What to do when eye contact makes your coworker uncomfortable?

Why does the Sun have different day lengths, but not the gas giants?

awk assign to multiple variables at once

Has the laser at Magurele, Romania reached a tenth of the Sun's power?

Why is it that I can sometimes guess the next note?

How to preserve electronics (computers, iPads and phones) for hundreds of years

"It doesn't matter" or "it won't matter"?

What does Apple's new App Store requirement mean

Which was the first story featuring espers?

Mimic lecturing on blackboard, facing audience

Doesn't the system of the Supreme Court oppose justice?

Short story about a deaf man, who cuts people tongues

Has any country ever had 2 former presidents in jail simultaneously?

Is it allowed to activate the ability of multiple planeswalkers in a single turn?

Is there any evidence that Cleopatra and Caesarion considered fleeing to India to escape the Romans?

Does the Linux kernel need a file system to run?

Why do Radio Buttons not fill the entire outer circle?

Why does AES have exactly 10 rounds for a 128-bit key, 12 for 192 bits and 14 for a 256-bit key size?



Showing a sum is positive


Finding Binomial expansion of a radicalSimplify the Expression $sum _ k=0 ^ n binomnki^k3^k-n $Alternative combinatorial proof for $sumlimits_r=0^nbinomnrbinomm+rn=sumlimits_r=0^nbinomnrbinommr2^r$Proof by induction, binomial coefficientApproximating a binomial sum over a simplexHow to expand $sqrtx^6+1$ using Maclaurin's seriesSum of $m choose j$ multiplied by $2^2^j$How to show that $sumlimits_k=0^n (-1)^ktfrac nchoosek x+kchoosek = fracxx+n$Finite sum with inverse binomialShowing an alternating sum is positive













2












$begingroup$



Show that the sum$$sum_k=0^n n choose kfrac(-1)^kn+k+1$$ is a positive rational number.




It is easy to show that it is a rational number. But I am having trouble showing that this expression is positive. It might be some binomial expansion that I could not get.










share|cite|improve this question











$endgroup$







  • 1




    $begingroup$
    Have you tried using induction on $n$ for example?
    $endgroup$
    – Minus One-Twelfth
    1 hour ago
















2












$begingroup$



Show that the sum$$sum_k=0^n n choose kfrac(-1)^kn+k+1$$ is a positive rational number.




It is easy to show that it is a rational number. But I am having trouble showing that this expression is positive. It might be some binomial expansion that I could not get.










share|cite|improve this question











$endgroup$







  • 1




    $begingroup$
    Have you tried using induction on $n$ for example?
    $endgroup$
    – Minus One-Twelfth
    1 hour ago














2












2








2


1



$begingroup$



Show that the sum$$sum_k=0^n n choose kfrac(-1)^kn+k+1$$ is a positive rational number.




It is easy to show that it is a rational number. But I am having trouble showing that this expression is positive. It might be some binomial expansion that I could not get.










share|cite|improve this question











$endgroup$





Show that the sum$$sum_k=0^n n choose kfrac(-1)^kn+k+1$$ is a positive rational number.




It is easy to show that it is a rational number. But I am having trouble showing that this expression is positive. It might be some binomial expansion that I could not get.







combinatorics summation binomial-coefficients binomial-ideals






share|cite|improve this question















share|cite|improve this question













share|cite|improve this question




share|cite|improve this question








edited 1 hour ago







Hitendra Kumar

















asked 1 hour ago









Hitendra KumarHitendra Kumar

606




606







  • 1




    $begingroup$
    Have you tried using induction on $n$ for example?
    $endgroup$
    – Minus One-Twelfth
    1 hour ago













  • 1




    $begingroup$
    Have you tried using induction on $n$ for example?
    $endgroup$
    – Minus One-Twelfth
    1 hour ago








1




1




$begingroup$
Have you tried using induction on $n$ for example?
$endgroup$
– Minus One-Twelfth
1 hour ago





$begingroup$
Have you tried using induction on $n$ for example?
$endgroup$
– Minus One-Twelfth
1 hour ago











2 Answers
2






active

oldest

votes


















4












$begingroup$

Direct proof:
$$beginsplit
sum_k=0^n nchoose kfrac(-1)^kn+k+1 &=sum_k=0^n nchoose k(-1)^kint_0^1 x^n+kdx\
&=int_0^1x^nsum_k=0^n nchoose k(-x)^kdx\
&=int_0^1x^n(1-x)^ndx
endsplit$$

The latter is clearly a positive number.






share|cite|improve this answer









$endgroup$












  • $begingroup$
    Thanks,I got it.
    $endgroup$
    – Hitendra Kumar
    35 mins ago










  • $begingroup$
    You're welcome!
    $endgroup$
    – Stefan Lafon
    34 mins ago










  • $begingroup$
    How did you conclude that the sum is a limited integral? Do you know where can I find more on this on-line? Thanks.
    $endgroup$
    – NoChance
    34 mins ago






  • 1




    $begingroup$
    It's a "known trick" that $frac 1 p+1 = int_0^1x^pdx$. Then I noticed that the sum looked almost like that of the binomial theorem.
    $endgroup$
    – Stefan Lafon
    28 mins ago










  • $begingroup$
    Thanks for responding.Got it.
    $endgroup$
    – NoChance
    18 mins ago



















3












$begingroup$

When $k=0$ the term is positive. When $k=1$ the term is negative BUT SMALLER (in absolute value) THAN THE $k=0$ TERM.



When $k=2$ the term is positive. When $k=3$ the term is negative BUT SMALLER (in absolute value) THAN THE $k=2$ TERM.



.....



Get it?






share|cite|improve this answer









$endgroup$












  • $begingroup$
    sorry, I did not write question correctly. Now, I have corrected that. By looking at your answer I realized my mistake. Thanks
    $endgroup$
    – Hitendra Kumar
    1 hour ago










Your Answer





StackExchange.ifUsing("editor", function ()
return StackExchange.using("mathjaxEditing", function ()
StackExchange.MarkdownEditor.creationCallbacks.add(function (editor, postfix)
StackExchange.mathjaxEditing.prepareWmdForMathJax(editor, postfix, [["$", "$"], ["\\(","\\)"]]);
);
);
, "mathjax-editing");

StackExchange.ready(function()
var channelOptions =
tags: "".split(" "),
id: "69"
;
initTagRenderer("".split(" "), "".split(" "), channelOptions);

StackExchange.using("externalEditor", function()
// Have to fire editor after snippets, if snippets enabled
if (StackExchange.settings.snippets.snippetsEnabled)
StackExchange.using("snippets", function()
createEditor();
);

else
createEditor();

);

function createEditor()
StackExchange.prepareEditor(
heartbeatType: 'answer',
autoActivateHeartbeat: false,
convertImagesToLinks: true,
noModals: true,
showLowRepImageUploadWarning: true,
reputationToPostImages: 10,
bindNavPrevention: true,
postfix: "",
imageUploader:
brandingHtml: "Powered by u003ca class="icon-imgur-white" href="https://imgur.com/"u003eu003c/au003e",
contentPolicyHtml: "User contributions licensed under u003ca href="https://creativecommons.org/licenses/by-sa/3.0/"u003ecc by-sa 3.0 with attribution requiredu003c/au003e u003ca href="https://stackoverflow.com/legal/content-policy"u003e(content policy)u003c/au003e",
allowUrls: true
,
noCode: true, onDemand: true,
discardSelector: ".discard-answer"
,immediatelyShowMarkdownHelp:true
);



);













draft saved

draft discarded


















StackExchange.ready(
function ()
StackExchange.openid.initPostLogin('.new-post-login', 'https%3a%2f%2fmath.stackexchange.com%2fquestions%2f3157740%2fshowing-a-sum-is-positive%23new-answer', 'question_page');

);

Post as a guest















Required, but never shown

























2 Answers
2






active

oldest

votes








2 Answers
2






active

oldest

votes









active

oldest

votes






active

oldest

votes









4












$begingroup$

Direct proof:
$$beginsplit
sum_k=0^n nchoose kfrac(-1)^kn+k+1 &=sum_k=0^n nchoose k(-1)^kint_0^1 x^n+kdx\
&=int_0^1x^nsum_k=0^n nchoose k(-x)^kdx\
&=int_0^1x^n(1-x)^ndx
endsplit$$

The latter is clearly a positive number.






share|cite|improve this answer









$endgroup$












  • $begingroup$
    Thanks,I got it.
    $endgroup$
    – Hitendra Kumar
    35 mins ago










  • $begingroup$
    You're welcome!
    $endgroup$
    – Stefan Lafon
    34 mins ago










  • $begingroup$
    How did you conclude that the sum is a limited integral? Do you know where can I find more on this on-line? Thanks.
    $endgroup$
    – NoChance
    34 mins ago






  • 1




    $begingroup$
    It's a "known trick" that $frac 1 p+1 = int_0^1x^pdx$. Then I noticed that the sum looked almost like that of the binomial theorem.
    $endgroup$
    – Stefan Lafon
    28 mins ago










  • $begingroup$
    Thanks for responding.Got it.
    $endgroup$
    – NoChance
    18 mins ago
















4












$begingroup$

Direct proof:
$$beginsplit
sum_k=0^n nchoose kfrac(-1)^kn+k+1 &=sum_k=0^n nchoose k(-1)^kint_0^1 x^n+kdx\
&=int_0^1x^nsum_k=0^n nchoose k(-x)^kdx\
&=int_0^1x^n(1-x)^ndx
endsplit$$

The latter is clearly a positive number.






share|cite|improve this answer









$endgroup$












  • $begingroup$
    Thanks,I got it.
    $endgroup$
    – Hitendra Kumar
    35 mins ago










  • $begingroup$
    You're welcome!
    $endgroup$
    – Stefan Lafon
    34 mins ago










  • $begingroup$
    How did you conclude that the sum is a limited integral? Do you know where can I find more on this on-line? Thanks.
    $endgroup$
    – NoChance
    34 mins ago






  • 1




    $begingroup$
    It's a "known trick" that $frac 1 p+1 = int_0^1x^pdx$. Then I noticed that the sum looked almost like that of the binomial theorem.
    $endgroup$
    – Stefan Lafon
    28 mins ago










  • $begingroup$
    Thanks for responding.Got it.
    $endgroup$
    – NoChance
    18 mins ago














4












4








4





$begingroup$

Direct proof:
$$beginsplit
sum_k=0^n nchoose kfrac(-1)^kn+k+1 &=sum_k=0^n nchoose k(-1)^kint_0^1 x^n+kdx\
&=int_0^1x^nsum_k=0^n nchoose k(-x)^kdx\
&=int_0^1x^n(1-x)^ndx
endsplit$$

The latter is clearly a positive number.






share|cite|improve this answer









$endgroup$



Direct proof:
$$beginsplit
sum_k=0^n nchoose kfrac(-1)^kn+k+1 &=sum_k=0^n nchoose k(-1)^kint_0^1 x^n+kdx\
&=int_0^1x^nsum_k=0^n nchoose k(-x)^kdx\
&=int_0^1x^n(1-x)^ndx
endsplit$$

The latter is clearly a positive number.







share|cite|improve this answer












share|cite|improve this answer



share|cite|improve this answer










answered 1 hour ago









Stefan LafonStefan Lafon

3,00019




3,00019











  • $begingroup$
    Thanks,I got it.
    $endgroup$
    – Hitendra Kumar
    35 mins ago










  • $begingroup$
    You're welcome!
    $endgroup$
    – Stefan Lafon
    34 mins ago










  • $begingroup$
    How did you conclude that the sum is a limited integral? Do you know where can I find more on this on-line? Thanks.
    $endgroup$
    – NoChance
    34 mins ago






  • 1




    $begingroup$
    It's a "known trick" that $frac 1 p+1 = int_0^1x^pdx$. Then I noticed that the sum looked almost like that of the binomial theorem.
    $endgroup$
    – Stefan Lafon
    28 mins ago










  • $begingroup$
    Thanks for responding.Got it.
    $endgroup$
    – NoChance
    18 mins ago

















  • $begingroup$
    Thanks,I got it.
    $endgroup$
    – Hitendra Kumar
    35 mins ago










  • $begingroup$
    You're welcome!
    $endgroup$
    – Stefan Lafon
    34 mins ago










  • $begingroup$
    How did you conclude that the sum is a limited integral? Do you know where can I find more on this on-line? Thanks.
    $endgroup$
    – NoChance
    34 mins ago






  • 1




    $begingroup$
    It's a "known trick" that $frac 1 p+1 = int_0^1x^pdx$. Then I noticed that the sum looked almost like that of the binomial theorem.
    $endgroup$
    – Stefan Lafon
    28 mins ago










  • $begingroup$
    Thanks for responding.Got it.
    $endgroup$
    – NoChance
    18 mins ago
















$begingroup$
Thanks,I got it.
$endgroup$
– Hitendra Kumar
35 mins ago




$begingroup$
Thanks,I got it.
$endgroup$
– Hitendra Kumar
35 mins ago












$begingroup$
You're welcome!
$endgroup$
– Stefan Lafon
34 mins ago




$begingroup$
You're welcome!
$endgroup$
– Stefan Lafon
34 mins ago












$begingroup$
How did you conclude that the sum is a limited integral? Do you know where can I find more on this on-line? Thanks.
$endgroup$
– NoChance
34 mins ago




$begingroup$
How did you conclude that the sum is a limited integral? Do you know where can I find more on this on-line? Thanks.
$endgroup$
– NoChance
34 mins ago




1




1




$begingroup$
It's a "known trick" that $frac 1 p+1 = int_0^1x^pdx$. Then I noticed that the sum looked almost like that of the binomial theorem.
$endgroup$
– Stefan Lafon
28 mins ago




$begingroup$
It's a "known trick" that $frac 1 p+1 = int_0^1x^pdx$. Then I noticed that the sum looked almost like that of the binomial theorem.
$endgroup$
– Stefan Lafon
28 mins ago












$begingroup$
Thanks for responding.Got it.
$endgroup$
– NoChance
18 mins ago





$begingroup$
Thanks for responding.Got it.
$endgroup$
– NoChance
18 mins ago












3












$begingroup$

When $k=0$ the term is positive. When $k=1$ the term is negative BUT SMALLER (in absolute value) THAN THE $k=0$ TERM.



When $k=2$ the term is positive. When $k=3$ the term is negative BUT SMALLER (in absolute value) THAN THE $k=2$ TERM.



.....



Get it?






share|cite|improve this answer









$endgroup$












  • $begingroup$
    sorry, I did not write question correctly. Now, I have corrected that. By looking at your answer I realized my mistake. Thanks
    $endgroup$
    – Hitendra Kumar
    1 hour ago















3












$begingroup$

When $k=0$ the term is positive. When $k=1$ the term is negative BUT SMALLER (in absolute value) THAN THE $k=0$ TERM.



When $k=2$ the term is positive. When $k=3$ the term is negative BUT SMALLER (in absolute value) THAN THE $k=2$ TERM.



.....



Get it?






share|cite|improve this answer









$endgroup$












  • $begingroup$
    sorry, I did not write question correctly. Now, I have corrected that. By looking at your answer I realized my mistake. Thanks
    $endgroup$
    – Hitendra Kumar
    1 hour ago













3












3








3





$begingroup$

When $k=0$ the term is positive. When $k=1$ the term is negative BUT SMALLER (in absolute value) THAN THE $k=0$ TERM.



When $k=2$ the term is positive. When $k=3$ the term is negative BUT SMALLER (in absolute value) THAN THE $k=2$ TERM.



.....



Get it?






share|cite|improve this answer









$endgroup$



When $k=0$ the term is positive. When $k=1$ the term is negative BUT SMALLER (in absolute value) THAN THE $k=0$ TERM.



When $k=2$ the term is positive. When $k=3$ the term is negative BUT SMALLER (in absolute value) THAN THE $k=2$ TERM.



.....



Get it?







share|cite|improve this answer












share|cite|improve this answer



share|cite|improve this answer










answered 1 hour ago









David G. StorkDavid G. Stork

11.1k41432




11.1k41432











  • $begingroup$
    sorry, I did not write question correctly. Now, I have corrected that. By looking at your answer I realized my mistake. Thanks
    $endgroup$
    – Hitendra Kumar
    1 hour ago
















  • $begingroup$
    sorry, I did not write question correctly. Now, I have corrected that. By looking at your answer I realized my mistake. Thanks
    $endgroup$
    – Hitendra Kumar
    1 hour ago















$begingroup$
sorry, I did not write question correctly. Now, I have corrected that. By looking at your answer I realized my mistake. Thanks
$endgroup$
– Hitendra Kumar
1 hour ago




$begingroup$
sorry, I did not write question correctly. Now, I have corrected that. By looking at your answer I realized my mistake. Thanks
$endgroup$
– Hitendra Kumar
1 hour ago

















draft saved

draft discarded
















































Thanks for contributing an answer to Mathematics Stack Exchange!


  • Please be sure to answer the question. Provide details and share your research!

But avoid


  • Asking for help, clarification, or responding to other answers.

  • Making statements based on opinion; back them up with references or personal experience.

Use MathJax to format equations. MathJax reference.


To learn more, see our tips on writing great answers.




draft saved


draft discarded














StackExchange.ready(
function ()
StackExchange.openid.initPostLogin('.new-post-login', 'https%3a%2f%2fmath.stackexchange.com%2fquestions%2f3157740%2fshowing-a-sum-is-positive%23new-answer', 'question_page');

);

Post as a guest















Required, but never shown





















































Required, but never shown














Required, but never shown












Required, but never shown







Required, but never shown

































Required, but never shown














Required, but never shown












Required, but never shown







Required, but never shown







Popular posts from this blog

कुँवर स्रोत दिक्चालन सूची"कुँवर""राणा कुँवरके वंशावली"

Why is a white electrical wire connected to 2 black wires?How to wire a light fixture with 3 white wires in box?How should I wire a ceiling fan when there's only three wires in the box?Two white, two black, two ground, and red wire in ceiling box connected to switchWhy is there a white wire connected to multiple black wires in my light box?How to wire a light with two white wires and one black wireReplace light switch connected to a power outlet with dimmer - two black wires to one black and redHow to wire a light with multiple black/white/green wires from the ceiling?Ceiling box has 2 black and white wires but fan/ light only has 1 of eachWhy neutral wire connected to load wire?Switch with 2 black, 2 white, 2 ground and 1 red wire connected to ceiling light and a receptacle?

चैत्य भूमि चित्र दीर्घा सन्दर्भ बाहरी कडियाँ दिक्चालन सूची"Chaitya Bhoomi""Chaitya Bhoomi: Statue of Equality in India""Dadar Chaitya Bhoomi: Statue of Equality in India""Ambedkar memorial: Centre okays transfer of Indu Mill land"चैत्यभमि